LSAT and Law School Admissions Forum

Get expert LSAT preparation and law school admissions advice from PowerScore Test Preparation.

 srcline@noctrl.edu
  • Posts: 243
  • Joined: Oct 16, 2015
|
#23706
Hello,

So not understanding this question at all. Can someone please explain why E is the correct answer. Where in the authors argument do they : generalize on the basis of what could be exceptional cases? The author does generalize b/c they are saying that no wealthy people should be on the GP committee b/c a number of G's wealthiest citizens have been criminals. But I'm not seeing the exceptional cases part.

Answer choices A and B are also really confusing for me.

Thankyou
Sarah
 David Boyle
PowerScore Staff
  • PowerScore Staff
  • Posts: 836
  • Joined: Jun 07, 2013
|
#23795
srcline@noctrl.edu wrote:Hello,

So not understanding this question at all. Can someone please explain why E is the correct answer. Where in the authors argument do they : generalize on the basis of what could be exceptional cases? The author does generalize b/c they are saying that no wealthy people should be on the GP committee b/c a number of G's wealthiest citizens have been criminals. But I'm not seeing the exceptional cases part.

Answer choices A and B are also really confusing for me.

Thankyou
Sarah

Hello,

The criminals could be just a few of the wealthy people, not all of them.
A and B should be confusing, since they don't really help at all.

David
 srcline@noctrl.edu
  • Posts: 243
  • Joined: Oct 16, 2015
|
#23929
Hello David,

Thankyou for your explanations. But I'm still a bit confused by this. My issue with answer choice E is the use of the words "exceptional cases". The author never says that they are exceptional cases.

Thankyou
Sarah
 Robert Carroll
PowerScore Staff
  • PowerScore Staff
  • Posts: 1787
  • Joined: Dec 06, 2013
|
#24109
Sarah,

The author's reasoning was flawed. The author wouldn't say this, though! The author thought the reasoning worked out ok, or else he/she would never have attempted this argument. Thus, we're identifying the flaw that the author missed (or didn't want us to pay any attention to!) - just because some wealthy citizens are criminals does not mean they all are. It is possible that the "some" citizens are exceptional cases - so we could create a committee of wealthy citizens that did not include those criminals. Thus the author's reasoning for excluding ALL wealthy people is mistaken.

Robert Carroll
User avatar
 PresidentLSAT
  • Posts: 87
  • Joined: Apr 19, 2021
|
#90317
Hello Powerscore,

I got this question right but was confused by A.

Would A have been close to identifying the flaw if the stimulus concluded that, the wealthy people on the committee are criminals?

My reason for choosing E was that the author generalized on the body of the committee based on instances where the wealthy people on it comprised of criminals.

Many thanks
 Robert Carroll
PowerScore Staff
  • PowerScore Staff
  • Posts: 1787
  • Joined: Dec 06, 2013
|
#90444
President,

Your reasoning for answer choice (E) is perfect.

Answer choice (A) has causal language in it, so this could be the flaw only if the stimulus contained causal reasoning. There is none there, so this isn't correct. If the conclusion was that the wealthy people on the committee are criminals, that still wouldn't involve causation.

Robert Carroll

Get the most out of your LSAT Prep Plus subscription.

Analyze and track your performance with our Testing and Analytics Package.